How many combinations to put books in $n$ packages - Whats wrong with my solutionHow many ways to divide group of 12 people into 2 groups of 3 people and 3 groups of 2 people?Combinatorial optimization - improve performanceCombinatorics homeworkcombinatorics shelf arrangementHow many different combinations of paths are there in this situation?Is there a tool to generate all combinations $C$ where $k=7$, $n=36$?How many functions are there from [9] to [7] if every image in the codomain has 3 arguments in the domain?Ways to arrange booksHow many combinations of sets / algorithm to generate all possible?Problem about eight different books randomly put on shelf.

Can an x86 CPU running in real mode be considered to be basically an 8086 CPU?

Perform and show arithmetic with LuaLaTeX

Could an aircraft fly or hover using only jets of compressed air?

What does it mean to describe someone as a butt steak?

Is it possible to run Internet Explorer on OS X El Capitan?

How does quantile regression compare to logistic regression with the variable split at the quantile?

What is the word for reserving something for yourself before others do?

Alternative to sending password over mail?

NMaximize is not converging to a solution

Why doesn't Newton's third law mean a person bounces back to where they started when they hit the ground?

How to format long polynomial?

meaning of に in 本当に?

What defenses are there against being summoned by the Gate spell?

What's the output of a record needle playing an out-of-speed record

dbcc cleantable batch size explanation

Can a monk's single staff be considered dual wielded, as per the Dual Wielder feat?

Why are electrically insulating heatsinks so rare? Is it just cost?

Cross compiling for RPi - error while loading shared libraries

Can I make popcorn with any corn?

How to source a part of a file

Why is 150k or 200k jobs considered good when there's 300k+ births a month?

DC-DC converter from low voltage at high current, to high voltage at low current

Roll the carpet

Why can't we play rap on piano?



How many combinations to put books in $n$ packages - Whats wrong with my solution


How many ways to divide group of 12 people into 2 groups of 3 people and 3 groups of 2 people?Combinatorial optimization - improve performanceCombinatorics homeworkcombinatorics shelf arrangementHow many different combinations of paths are there in this situation?Is there a tool to generate all combinations $C$ where $k=7$, $n=36$?How many functions are there from [9] to [7] if every image in the codomain has 3 arguments in the domain?Ways to arrange booksHow many combinations of sets / algorithm to generate all possible?Problem about eight different books randomly put on shelf.













0












$begingroup$


There are $2n$ science fiction books, $3n$ thrillers and $5n$ drama.



We want to fix $n$ packages of books so that in each package there will be 2 sci-fi, 3 thrillers and 5 drama.



How many combinations are to do that?



I thought about organising let's say the sci-fi books in a row $(2n)!$ and take every 2 as one, so putting in order in $n$ packages is $n!$.
Same with the other genres so that the solution is $$(2n)!n!(3n)!n!(5n)!n!$$
But not sure if it's the right solution.



What do you think?
How can I know if this is a correct approach?




Correct me if I'm wrong, I think my mistake is that I don't need the $(n!)^3)$ I wrote cuz it counts inside the $(2n)!(3n)!(5n)!$,
But I do need to divide with the inner order of each group of 2's, 3's and 5's means divide by $(2!)^n(3!)^(5!)^n$










share|cite|improve this question











$endgroup$







  • 1




    $begingroup$
    In your edit you are correct if the packages are ordered, but I suspect they are not. Then division with $n!$ repairs that.
    $endgroup$
    – drhab
    Mar 29 at 13:30















0












$begingroup$


There are $2n$ science fiction books, $3n$ thrillers and $5n$ drama.



We want to fix $n$ packages of books so that in each package there will be 2 sci-fi, 3 thrillers and 5 drama.



How many combinations are to do that?



I thought about organising let's say the sci-fi books in a row $(2n)!$ and take every 2 as one, so putting in order in $n$ packages is $n!$.
Same with the other genres so that the solution is $$(2n)!n!(3n)!n!(5n)!n!$$
But not sure if it's the right solution.



What do you think?
How can I know if this is a correct approach?




Correct me if I'm wrong, I think my mistake is that I don't need the $(n!)^3)$ I wrote cuz it counts inside the $(2n)!(3n)!(5n)!$,
But I do need to divide with the inner order of each group of 2's, 3's and 5's means divide by $(2!)^n(3!)^(5!)^n$










share|cite|improve this question











$endgroup$







  • 1




    $begingroup$
    In your edit you are correct if the packages are ordered, but I suspect they are not. Then division with $n!$ repairs that.
    $endgroup$
    – drhab
    Mar 29 at 13:30













0












0








0





$begingroup$


There are $2n$ science fiction books, $3n$ thrillers and $5n$ drama.



We want to fix $n$ packages of books so that in each package there will be 2 sci-fi, 3 thrillers and 5 drama.



How many combinations are to do that?



I thought about organising let's say the sci-fi books in a row $(2n)!$ and take every 2 as one, so putting in order in $n$ packages is $n!$.
Same with the other genres so that the solution is $$(2n)!n!(3n)!n!(5n)!n!$$
But not sure if it's the right solution.



What do you think?
How can I know if this is a correct approach?




Correct me if I'm wrong, I think my mistake is that I don't need the $(n!)^3)$ I wrote cuz it counts inside the $(2n)!(3n)!(5n)!$,
But I do need to divide with the inner order of each group of 2's, 3's and 5's means divide by $(2!)^n(3!)^(5!)^n$










share|cite|improve this question











$endgroup$




There are $2n$ science fiction books, $3n$ thrillers and $5n$ drama.



We want to fix $n$ packages of books so that in each package there will be 2 sci-fi, 3 thrillers and 5 drama.



How many combinations are to do that?



I thought about organising let's say the sci-fi books in a row $(2n)!$ and take every 2 as one, so putting in order in $n$ packages is $n!$.
Same with the other genres so that the solution is $$(2n)!n!(3n)!n!(5n)!n!$$
But not sure if it's the right solution.



What do you think?
How can I know if this is a correct approach?




Correct me if I'm wrong, I think my mistake is that I don't need the $(n!)^3)$ I wrote cuz it counts inside the $(2n)!(3n)!(5n)!$,
But I do need to divide with the inner order of each group of 2's, 3's and 5's means divide by $(2!)^n(3!)^(5!)^n$







combinatorics






share|cite|improve this question















share|cite|improve this question













share|cite|improve this question




share|cite|improve this question








edited Mar 29 at 12:46







Evy

















asked Mar 29 at 10:25









EvyEvy

244




244







  • 1




    $begingroup$
    In your edit you are correct if the packages are ordered, but I suspect they are not. Then division with $n!$ repairs that.
    $endgroup$
    – drhab
    Mar 29 at 13:30












  • 1




    $begingroup$
    In your edit you are correct if the packages are ordered, but I suspect they are not. Then division with $n!$ repairs that.
    $endgroup$
    – drhab
    Mar 29 at 13:30







1




1




$begingroup$
In your edit you are correct if the packages are ordered, but I suspect they are not. Then division with $n!$ repairs that.
$endgroup$
– drhab
Mar 29 at 13:30




$begingroup$
In your edit you are correct if the packages are ordered, but I suspect they are not. Then division with $n!$ repairs that.
$endgroup$
– drhab
Mar 29 at 13:30










1 Answer
1






active

oldest

votes


















4












$begingroup$

Let us first label the packings by $1,2,dots n$.



We start with selecting $2$ sci-fi's, $3$ thrillers and $5$ drama's meant for the first package.



For that there are $binom2n2binom3n3binom5n5$ possibilities.



Then for the second package there are $binom2n-22binom3n-33binom5n-55$ possibilities et cetera, so - if the order of the packages counts - then there are: $$binom2n2binom3n3binom5n5timesbinom2n-22binom3n-33binom5n-55timescdotstimesbinom22binom33binom55=$$$$frac(2n)!(3n)!(5n)!(2!)^n(3!)^n(5!)^n=frac(2n)!(3n)!(5n)!1440^n$$possibilities.



If the order of the packages does not count (and I suspect that is the case here) then every possibility has been counted $n!$ times above, and repairing this we find:$$frac(2n)!(3n)!(5n)!n!1440^n$$possibilities.






share|cite|improve this answer











$endgroup$












  • $begingroup$
    Thank you, can you please explain what's wrong with my solution
    $endgroup$
    – Evy
    Mar 29 at 11:12










  • $begingroup$
    Also, Should'nt it be $(2!)^n(3!)^n(5!)^n$
    $endgroup$
    – Evy
    Mar 29 at 12:10











  • $begingroup$
    Yes, that is what it should be (I repaired).
    $endgroup$
    – drhab
    Mar 29 at 13:15












Your Answer





StackExchange.ifUsing("editor", function ()
return StackExchange.using("mathjaxEditing", function ()
StackExchange.MarkdownEditor.creationCallbacks.add(function (editor, postfix)
StackExchange.mathjaxEditing.prepareWmdForMathJax(editor, postfix, [["$", "$"], ["\\(","\\)"]]);
);
);
, "mathjax-editing");

StackExchange.ready(function()
var channelOptions =
tags: "".split(" "),
id: "69"
;
initTagRenderer("".split(" "), "".split(" "), channelOptions);

StackExchange.using("externalEditor", function()
// Have to fire editor after snippets, if snippets enabled
if (StackExchange.settings.snippets.snippetsEnabled)
StackExchange.using("snippets", function()
createEditor();
);

else
createEditor();

);

function createEditor()
StackExchange.prepareEditor(
heartbeatType: 'answer',
autoActivateHeartbeat: false,
convertImagesToLinks: true,
noModals: true,
showLowRepImageUploadWarning: true,
reputationToPostImages: 10,
bindNavPrevention: true,
postfix: "",
imageUploader:
brandingHtml: "Powered by u003ca class="icon-imgur-white" href="https://imgur.com/"u003eu003c/au003e",
contentPolicyHtml: "User contributions licensed under u003ca href="https://creativecommons.org/licenses/by-sa/3.0/"u003ecc by-sa 3.0 with attribution requiredu003c/au003e u003ca href="https://stackoverflow.com/legal/content-policy"u003e(content policy)u003c/au003e",
allowUrls: true
,
noCode: true, onDemand: true,
discardSelector: ".discard-answer"
,immediatelyShowMarkdownHelp:true
);



);













draft saved

draft discarded


















StackExchange.ready(
function ()
StackExchange.openid.initPostLogin('.new-post-login', 'https%3a%2f%2fmath.stackexchange.com%2fquestions%2f3166979%2fhow-many-combinations-to-put-books-in-n-packages-whats-wrong-with-my-solutio%23new-answer', 'question_page');

);

Post as a guest















Required, but never shown

























1 Answer
1






active

oldest

votes








1 Answer
1






active

oldest

votes









active

oldest

votes






active

oldest

votes









4












$begingroup$

Let us first label the packings by $1,2,dots n$.



We start with selecting $2$ sci-fi's, $3$ thrillers and $5$ drama's meant for the first package.



For that there are $binom2n2binom3n3binom5n5$ possibilities.



Then for the second package there are $binom2n-22binom3n-33binom5n-55$ possibilities et cetera, so - if the order of the packages counts - then there are: $$binom2n2binom3n3binom5n5timesbinom2n-22binom3n-33binom5n-55timescdotstimesbinom22binom33binom55=$$$$frac(2n)!(3n)!(5n)!(2!)^n(3!)^n(5!)^n=frac(2n)!(3n)!(5n)!1440^n$$possibilities.



If the order of the packages does not count (and I suspect that is the case here) then every possibility has been counted $n!$ times above, and repairing this we find:$$frac(2n)!(3n)!(5n)!n!1440^n$$possibilities.






share|cite|improve this answer











$endgroup$












  • $begingroup$
    Thank you, can you please explain what's wrong with my solution
    $endgroup$
    – Evy
    Mar 29 at 11:12










  • $begingroup$
    Also, Should'nt it be $(2!)^n(3!)^n(5!)^n$
    $endgroup$
    – Evy
    Mar 29 at 12:10











  • $begingroup$
    Yes, that is what it should be (I repaired).
    $endgroup$
    – drhab
    Mar 29 at 13:15
















4












$begingroup$

Let us first label the packings by $1,2,dots n$.



We start with selecting $2$ sci-fi's, $3$ thrillers and $5$ drama's meant for the first package.



For that there are $binom2n2binom3n3binom5n5$ possibilities.



Then for the second package there are $binom2n-22binom3n-33binom5n-55$ possibilities et cetera, so - if the order of the packages counts - then there are: $$binom2n2binom3n3binom5n5timesbinom2n-22binom3n-33binom5n-55timescdotstimesbinom22binom33binom55=$$$$frac(2n)!(3n)!(5n)!(2!)^n(3!)^n(5!)^n=frac(2n)!(3n)!(5n)!1440^n$$possibilities.



If the order of the packages does not count (and I suspect that is the case here) then every possibility has been counted $n!$ times above, and repairing this we find:$$frac(2n)!(3n)!(5n)!n!1440^n$$possibilities.






share|cite|improve this answer











$endgroup$












  • $begingroup$
    Thank you, can you please explain what's wrong with my solution
    $endgroup$
    – Evy
    Mar 29 at 11:12










  • $begingroup$
    Also, Should'nt it be $(2!)^n(3!)^n(5!)^n$
    $endgroup$
    – Evy
    Mar 29 at 12:10











  • $begingroup$
    Yes, that is what it should be (I repaired).
    $endgroup$
    – drhab
    Mar 29 at 13:15














4












4








4





$begingroup$

Let us first label the packings by $1,2,dots n$.



We start with selecting $2$ sci-fi's, $3$ thrillers and $5$ drama's meant for the first package.



For that there are $binom2n2binom3n3binom5n5$ possibilities.



Then for the second package there are $binom2n-22binom3n-33binom5n-55$ possibilities et cetera, so - if the order of the packages counts - then there are: $$binom2n2binom3n3binom5n5timesbinom2n-22binom3n-33binom5n-55timescdotstimesbinom22binom33binom55=$$$$frac(2n)!(3n)!(5n)!(2!)^n(3!)^n(5!)^n=frac(2n)!(3n)!(5n)!1440^n$$possibilities.



If the order of the packages does not count (and I suspect that is the case here) then every possibility has been counted $n!$ times above, and repairing this we find:$$frac(2n)!(3n)!(5n)!n!1440^n$$possibilities.






share|cite|improve this answer











$endgroup$



Let us first label the packings by $1,2,dots n$.



We start with selecting $2$ sci-fi's, $3$ thrillers and $5$ drama's meant for the first package.



For that there are $binom2n2binom3n3binom5n5$ possibilities.



Then for the second package there are $binom2n-22binom3n-33binom5n-55$ possibilities et cetera, so - if the order of the packages counts - then there are: $$binom2n2binom3n3binom5n5timesbinom2n-22binom3n-33binom5n-55timescdotstimesbinom22binom33binom55=$$$$frac(2n)!(3n)!(5n)!(2!)^n(3!)^n(5!)^n=frac(2n)!(3n)!(5n)!1440^n$$possibilities.



If the order of the packages does not count (and I suspect that is the case here) then every possibility has been counted $n!$ times above, and repairing this we find:$$frac(2n)!(3n)!(5n)!n!1440^n$$possibilities.







share|cite|improve this answer














share|cite|improve this answer



share|cite|improve this answer








edited Mar 29 at 13:10

























answered Mar 29 at 10:49









drhabdrhab

104k545136




104k545136











  • $begingroup$
    Thank you, can you please explain what's wrong with my solution
    $endgroup$
    – Evy
    Mar 29 at 11:12










  • $begingroup$
    Also, Should'nt it be $(2!)^n(3!)^n(5!)^n$
    $endgroup$
    – Evy
    Mar 29 at 12:10











  • $begingroup$
    Yes, that is what it should be (I repaired).
    $endgroup$
    – drhab
    Mar 29 at 13:15

















  • $begingroup$
    Thank you, can you please explain what's wrong with my solution
    $endgroup$
    – Evy
    Mar 29 at 11:12










  • $begingroup$
    Also, Should'nt it be $(2!)^n(3!)^n(5!)^n$
    $endgroup$
    – Evy
    Mar 29 at 12:10











  • $begingroup$
    Yes, that is what it should be (I repaired).
    $endgroup$
    – drhab
    Mar 29 at 13:15
















$begingroup$
Thank you, can you please explain what's wrong with my solution
$endgroup$
– Evy
Mar 29 at 11:12




$begingroup$
Thank you, can you please explain what's wrong with my solution
$endgroup$
– Evy
Mar 29 at 11:12












$begingroup$
Also, Should'nt it be $(2!)^n(3!)^n(5!)^n$
$endgroup$
– Evy
Mar 29 at 12:10





$begingroup$
Also, Should'nt it be $(2!)^n(3!)^n(5!)^n$
$endgroup$
– Evy
Mar 29 at 12:10













$begingroup$
Yes, that is what it should be (I repaired).
$endgroup$
– drhab
Mar 29 at 13:15





$begingroup$
Yes, that is what it should be (I repaired).
$endgroup$
– drhab
Mar 29 at 13:15


















draft saved

draft discarded
















































Thanks for contributing an answer to Mathematics Stack Exchange!


  • Please be sure to answer the question. Provide details and share your research!

But avoid


  • Asking for help, clarification, or responding to other answers.

  • Making statements based on opinion; back them up with references or personal experience.

Use MathJax to format equations. MathJax reference.


To learn more, see our tips on writing great answers.




draft saved


draft discarded














StackExchange.ready(
function ()
StackExchange.openid.initPostLogin('.new-post-login', 'https%3a%2f%2fmath.stackexchange.com%2fquestions%2f3166979%2fhow-many-combinations-to-put-books-in-n-packages-whats-wrong-with-my-solutio%23new-answer', 'question_page');

);

Post as a guest















Required, but never shown





















































Required, but never shown














Required, but never shown












Required, but never shown







Required, but never shown

































Required, but never shown














Required, but never shown












Required, but never shown







Required, but never shown







Popular posts from this blog

Triangular numbers and gcdProving sum of a set is $0 pmod n$ if $n$ is odd, or $fracn2 pmod n$ if $n$ is even?Is greatest common divisor of two numbers really their smallest linear combination?GCD, LCM RelationshipProve a set of nonnegative integers with greatest common divisor 1 and closed under addition has all but finite many nonnegative integers.all pairs of a and b in an equation containing gcdTriangular Numbers Modulo $k$ - Hit All Values?Understanding the Existence and Uniqueness of the GCDGCD and LCM with logical symbolsThe greatest common divisor of two positive integers less than 100 is equal to 3. Their least common multiple is twelve times one of the integers.Suppose that for all integers $x$, $x|a$ and $x|b$ if and only if $x|c$. Then $c = gcd(a,b)$Which is the gcd of 2 numbers which are multiplied and the result is 600000?

Ingelân Ynhâld Etymology | Geografy | Skiednis | Polityk en bestjoer | Ekonomy | Demografy | Kultuer | Klimaat | Sjoch ek | Keppelings om utens | Boarnen, noaten en referinsjes Navigaasjemenuwww.gov.ukOffisjele webside fan it regear fan it Feriene KeninkrykOffisjele webside fan it Britske FerkearsburoNederlânsktalige ynformaasje fan it Britske FerkearsburoOffisjele webside fan English Heritage, de organisaasje dy't him ynset foar it behâld fan it Ingelske kultuergoedYnwennertallen fan alle Britske stêden út 'e folkstelling fan 2011Notes en References, op dizze sideEngland

Հադիս Բովանդակություն Անվանում և նշանակություն | Դասակարգում | Աղբյուրներ | Նավարկման ցանկ